Masses and combined pulleys

  • Thread starter Karol
  • Start date
  • Tags
    Pulleys
In summary,1) Mass accelerates faster than g.2) What's m1's acceleration.3) In which condition m3 will move upwards, if they start from rest.4) If there is something wrong with the simple and straightforward analysis I presented in post #4, please point it out.5) Why don't you take into account ##\ddot y_2## in:$$a_3=-\left(\frac{d^2y_1}{dt^2}+\frac{d^2y_3}{dt^2}\right)$$In summary,1) Mass accelerates faster than
  • #1
Karol
1,380
22

Homework Statement


Snap1.jpg

1) Which mass accelerates faster than g
2) What's m1's acceleration
3) In which condition m3 will move upwards, if they start from rest

Homework Equations


Mass-acceleration: F=ma

The Attempt at a Solution


$$\left\{\begin{array}{l} (1)~a_3=2a_2+2a_1 \\ (2)~2T_3+m_2g-T_3=m_2a_2 \\ (3)~m_1g-2T_3=m_1a_1 \\ (4)~ T_3-m_3g=m_3a_3 \end{array}\right.$$
$$\rightarrow~a_2=\frac{m_1m_2+4m_2m_3+3m_1m_3}{m_1m_2+4m_2m_3-2m_1m_3}g$$
The fraction is always greater than 1, so a2 accelerates faster than g.
$$a_1=\frac{m_1m_2-6m_2m_3-2m_1m_3}{m_1m_2+4m_2m_3-2m_1m_3}g$$
3) eq' (1):
$$a_3=2a_2+2a_1~~\rightarrow~~a_3=\left( \frac{2m_1m_2-2m_2m_3+m_1m_3}{m_1m_2+4m_2m_3-2m_1m_3} \right)2g$$
I made eq' (4) as m3 ascending, so if a3 is positive-it ascends. from (5) two conditions are necessary, either both, the nominator and denominator are positive or are they negative. the first scenario:
$$\left\{ \begin{array}{l} (6)~2m_1m_2-2m_2m_3+m_1m_3>0 \\ (7)~m_1m_2+4m_2m_3-2m_1m_3>0 \end{array}\right.$$
From (6) i get (i multiply by 2): ##4m_1m_2+2m_1m_3>4m_2m_3##
From (7): ##m_1m_2+4m_2m_3>2m_1m_3##
I combine those 2 to get: ##5m_2>0##, but this is always true.
 
Physics news on Phys.org
  • #2
I don't match your kinematic equation (1).
 
  • Like
Likes TSny
  • #3
Chestermiller said:
I don't match your kinematic equation (1).
Snap1.jpg
The initial length of the rope equals to it's length after the change:
$$x_1+x_2+(x_1-x_2)+x_3=(x_2+\Delta_2)+(x_1+\Delta_1)+(x_1-x_2-\Delta_2+\Delta_1)+(x_3-\Delta_3)$$
$$\rightarrow~2\Delta_1=\Delta_3\rightarrow~2a_1=a_3$$
 
  • #4
Call ##y_1## the distance between the center of the upper pulley and the center of the middle pulley, ##y_2## the distance between the center of the middle pulley and the center of the bottom pulley, and ##y_3## the distance between the center of the middle pulley and mass 3. Then the total length of rope (excluding the parts wrapped on the pulleys, which are constant) is $$L=2y_1+2y_2 +y_3$$ Since the length L is constant, $$2\frac{d^2y_1}{dt^2}+2\frac{d^2y_2}{dt^2}+\frac{d^2y_3}{dt^2}=0$$In terms of these parameters, $$a_1=\frac{d^2y_1}{dt^2}+\frac{d^2y_2}{dt^2}$$
$$a_2=\frac{d^2y_1}{dt^2}$$
$$a_3=-\left(\frac{d^2y_1}{dt^2}+\frac{d^2y_3}{dt^2}\right)$$
From these equations, if follows that $$a_3=2a_1-a_2$$
 
Last edited:
  • #5
I worked the problem and I agree with Karol, concerning the constraint equation. I have a hard time believing the the delta l2 cancels out but I used graph paper and tried to draw the system and the system at a later time and I got Karol's constraint equation. Mass 2 still has greater than acceleration g.
 
  • #6
Karol said:
View attachment 106541 The initial length of the rope equals to it's length after the change:
$$x_1+x_2+(x_1-x_2)+x_3=(x_2+\Delta_2)+(x_1+\Delta_1)+(x_1-x_2-\Delta_2+\Delta_1)+(x_3-\Delta_3)$$
$$\rightarrow~2\Delta_1=\Delta_3\rightarrow~2a_1=a_3$$
You've defined ##x_3## as the displacement of ##m_3## from one of the moving pulleys. So, the acceleration determined by ##x_3## will be the acceleration of ##m_3## relative to that pulley rather than relative to the Earth frame of reference.
 
  • #7
mpresic said:
I worked the problem and I agree with Karol, concerning the constraint equation. I have a hard time believing the the delta l2 cancels out but I used graph paper and tried to draw the system and the system at a later time and I got Karol's constraint equation. Mass 2 still has greater than acceleration g.
Show us what you did. TSny already demonstrated an error in Karol's result, so this must be a common feature of yours as well.

If there is something wrong with the simple and straightforward analysis I presented in post #4, please point it out.
 
  • #8
Why don't you take into account ##\ddot y_2## in:
$$a_3=-\left(\frac{d^2y_1}{dt^2}+\frac{d^2y_3}{dt^2}\right)$$
 
  • #9
Karol said:
Why don't you take into account ##\ddot y_2## in:
$$a_3=-\left(\frac{d^2y_1}{dt^2}+\frac{d^2y_3}{dt^2}\right)$$
In post.#4, I meant to say that y3 is the distance between the middle pulley and m3. I've gone back and changed it.
 
Last edited:
  • #10
Chestermiller said:
In post.#4, I meant to say that y3 is the distance between the middle pulley and m3. I've gone back and changed it.
You didn't change anything, and it's like you've said in post #4:
Snap1.jpg
 
  • #11
Karol said:
You didn't change anything, and it's like you've said in post #4:
View attachment 106604
Yes, I did. I changed the word "lower" to "middle" in referring to y3. y3 is the distance between the center of the middle pulley and m3.
 
  • #12
Yes, that fixes it, thank you mpresic, TSny and Chestermiller.
But i forgot i didn't finish. m1's acceleration is ##a_1=\frac{a_3-a_2}{2}##
For m to accelerate upwards, from the relation: ##a_3=2a_1-a_2~~\rightarrow~~2a_1>a_2##
Now i will translate it to masses
 
Last edited:
  • #13
$$\left\{\begin{array}{l} (1)~a_3=2a_1-a_2 \\ (2)~2T_3+m_2g-T_3=m_2a_2 \\ (3)~m_1g-2T_3=m_1a_1 \\ (4)~ T_3-m_3g=m_3a_3 \end{array}\right.$$
$$\rightarrow~a_1=-\frac{m_3}{5m_2},~~a_3=\frac{m_3}{10m_2}-g$$
For m to move upwards, according to the direction in the drawing, a3 must be positive:
$$a_3>0~\rightarrow~~\frac{m_3}{10m_2}>g$$
 
  • #14
Karol said:
$$\left\{\begin{array}{l} (1)~a_3=2a_1-a_2 \\ (2)~2T_3+m_2g-T_3=m_2a_2 \\ (3)~m_1g-2T_3=m_1a_1 \\ (4)~ T_3-m_3g=m_3a_3 \end{array}\right.$$
$$\rightarrow~a_1=-\frac{m_3}{5m_2},~~a_3=\frac{m_3}{10m_2}-g$$
For m to move upwards, according to the direction in the drawing, a3 must be positive:
$$a_3>0~\rightarrow~~\frac{m_3}{10m_2}>g$$
That's not what I get.
 
  • #15
I agree with the 4 equations in post #14, but not with the results for a1 and a3.
 
  • Like
Likes Chestermiller
  • #16
I now agree with the four equations of Chestermiller, but I did have the wrong constraint equation earlier, I had a3 = 2 a1 (without a2). I looked at the length relative to the moving (pully), and not relative to the fixed (top) pulley. I now agree with the newest 4 equations
 
  • #17
TSny said:
I agree with the 4 equations in post #14, but not with the results for a1 and a3.
Thank you TSny,:
$$\left\{\begin{array}{l} (1)~a_3=2a_1-a_2 \\ (2)~2T_3+m_2g-T_3=m_2a_2 \\ (3)~m_1g-2T_3=m_1a_1 \\ (4)~ T_3-m_3g=m_3a_3 \end{array}\right.$$
$$(2)\& (4):~(5)~a_2=\frac{m_3}{m_2}a_3+\frac{m_3}{m_2}g+g$$
$$(3)\& (4):~(6)~a_3=\left( \frac{m_1-2m_3}{2m_3} \right)g-\frac{m_1}{2m_3}a_1$$
$$(1)\& (5)\& (6):~(7)~\left( \frac{m_1m_2+2m_3^2}{2m_2m_3} \right)g+\frac{m_3}{m_2}a_3=\left( 2+\frac{m_1}{2m_3} \right)a_1$$
$$(6)\& (7):~(8)~a_1=\frac{(m_2+m_3)g}{m_1m_2+m_1m_3+4m_2m_3}$$
$$(6)\& (8):~a_3=\frac{1}{2m_3}\left( m_1-2m_3-\frac{m_1(m2+m_3)}{m_1m_2+m_1m_3+4m_2m_3} \right)g$$
For m3 to move up it must be positive, in accordance with my drawing:
$$a_3>0~~\rightarrow~~m_1>2m_3+\frac{m_1(m_2+m_3)}{m_1m_2+m_1m_3+4m_2m_3}$$
 
  • #18
Karol said:
Thank you TSny,:
$$\left\{\begin{array}{l} (1)~a_3=2a_1-a_2 \\ (2)~2T_3+m_2g-T_3=m_2a_2 \\ (3)~m_1g-2T_3=m_1a_1 \\ (4)~ T_3-m_3g=m_3a_3 \end{array}\right.$$
$$(2)\& (4):~(5)~a_2=\frac{m_3}{m_2}a_3+\frac{m_3}{m_2}g+g$$
$$(3)\& (4):~(6)~a_3=\left( \frac{m_1-2m_3}{2m_3} \right)g-\frac{m_1}{2m_3}a_1$$
$$(1)\& (5)\& (6):~(7)~\left( \frac{m_1m_2+2m_3^2}{2m_2m_3} \right)g+\frac{m_3}{m_2}a_3=\left( 2+\frac{m_1}{2m_3} \right)a_1$$
$$(6)\& (7):~(8)~a_1=\frac{(m_2+m_3)g}{m_1m_2+m_1m_3+4m_2m_3}$$
$$(6)\& (8):~a_3=\frac{1}{2m_3}\left( m_1-2m_3-\frac{m_1(m2+m_3)}{m_1m_2+m_1m_3+4m_2m_3} \right)g$$
For m3 to move up it must be positive, in accordance with my drawing:
$$a_3>0~~\rightarrow~~m_1>2m_3+\frac{m_1(m_2+m_3)}{m_1m_2+m_1m_3+4m_2m_3}$$
Try again. The units don't match.
 
  • #19
Chestermiller said:
Try again. The units don't match.
$$\left\{\begin{array}{l} (1)~a_3=2a_1-a_2 \\ (2)~2T_3+m_2g-T_3=m_2a_2 \\ (3)~m_1g-2T_3=m_1a_1 \\ (4)~ T_3-m_3g=m_3a_3 \end{array}\right.$$
$$(2)\& (4):~(5)~a_2=\frac{m_3}{m_2}a_3+\left( 1+\frac{m_3}{m_2} \right)g$$
$$(3)\& (4):~(6)~a_3=\left( \frac{m_1}{2m_3}-1 \right)g-\frac{m_1}{2m_3}a_1$$
$$(1)\& (5)\& (6):~(7)~a_1=\left( \frac{m_1m_2+2m_3^2}{m_2(m_1+4m_3)} \right)g+\left( \frac{2m_3}{m_1+4m_3} \right)a_3$$
$$(6)\& (7):~(8)~a_3=\frac{m_1^2-2m_1m_3-10m_3^2-m_1m_2}{4m_3(m_1+2m_3)}$$
For m3 to move up it must be positive, in accordance with my drawing:
$$a_3>0~\rightarrow~m_1^2>2m_1m_3+m_1m_2+10m_3^2$$
 
  • #20
I get $$m_3<\frac{m_1m_2}{(m_1+4m_2)}$$
 
  • #21
$$\left\{\begin{array}{l} (1)~a_3=2a_1-a_2 \\ (2)~2T+m_2g-T=m_2a_2 \\ (3)~m_1g-2T=m_1a_1 \\ (4)~ T-m_3g=m_3a_3 \end{array}\right.$$
$$(1)\& (4):~(5)~~T-m_3g=2m_3a_1-m_3a_2$$
$$(2):~~T=m_2a_2-m_2g,~~(2)\& (5):~(6):~~2a_1=\left( \frac{m_2+m_3}{m_3} \right)a_2-\left( \frac{m_2+m_3}{m_3} \right)g$$
$$(2)\& (4)~:(7)~~a_2=\frac{m_3}{m_2}a_3+\left( 1+\frac{m_3}{m_2} \right)g$$
$$(3)\& (4):~(8)~~a_1=\left( 1-\frac{2m_3}{m_1} \right)g-\frac{2m_3}{m_1}a_3$$
$$(6)\& (7)\& (8):~(9)~~a_3=\frac{m_1m_2-m_1m_3-4m_2m_3}{m_1m_2+m_1m_3+4m_2m_3}g$$
$$a_3>0~\rightarrow~~m_3<\frac{m_1m_2}{m_1+4m_2}$$
Thank you Chestermiller
 
  • #22
Chestermiller said:
Call ##y_1## the distance between the center of the upper pulley and the center of the middle pulley, ##y_2## the distance between the center of the middle pulley and the center of the bottom pulley, and ##y_3## the distance between the center of the middle pulley and mass 3. Then the total length of rope (excluding the parts wrapped on the pulleys, which are constant) is $$L=2y_1+2y_2 +y_3$$ Since the length L is constant, $$2\frac{d^2y_1}{dt^2}+2\frac{d^2y_2}{dt^2}+\frac{d^2y_3}{dt^2}=0$$In terms of these parameters, $$a_1=\frac{d^2y_1}{dt^2}+\frac{d^2y_2}{dt^2}$$
$$a_2=\frac{d^2y_1}{dt^2}$$
$$a_3=-\left(\frac{d^2y_1}{dt^2}+\frac{d^2y_3}{dt^2}\right)$$
From these equations, if follows that $$a_3=2a_1-a_2$$

Hello ,

Let y1 be the distance of m1 from fixed pulley , y2 be distance of m2 and y3 be distance of m3 .Let h be fixed distance between m2 and middle pulley .

Since length of string is constant ,

y1 + y1 - y2 - h + y3 - y2 - h + y2 = constant

Differentiating twice I get ,

a3 = a2 - 2a1 . This has sign difference from the result you have obtained . I surely must be making a mistake somewhere .

Could you please tell my error .

Thanks
 
  • #23
Vibhor said:
a3 = a2 - 2a1 . This has sign difference from the result you have obtained .
In the original figure, ##a_3## is defined as positive in the upward direction while ##a_1## and ##a_2## are defined as positive in the downward direction.
 
  • Like
Likes Vibhor
  • #24
Oh !

In that case what I have done is correct assuming downward is positive for all the masses ?
 
  • #25
Vibhor said:
In that case what I have done is correct assuming downward is positive for all the masses ?
Yes. Looks good.
 
  • Like
Likes Vibhor
  • #26
TSny said:
Yes. Looks good.

Thanks .

You very well know I invariably have a hard time with moving pulleys :biggrin:
 
  • #27
In these types of problems of moving blocks and pulleys , or any other problems ( strings massless and pulley massless , frictionless ) from laws of motion chapter which is covered before rotational mechanics , we assume that pulleys rotate such that strings do not slip .

Suppose the pulley groove is frictionless such that string slips over it , does that change our result ?

I think the results remain same irrespective of whether string slips over the pulley or if pulley rotates along with string .

Am I correct ?

Thanks
 
  • #28
Vibhor said:
I think the results remain same irrespective of whether string slips over the pulley or if pulley rotates along with string .

Am I correct ?
Yes, as long as the pulleys are treated as massless and frictionless when they rotate.
 
  • Like
Likes Vibhor

1. What is a mass?

A mass is a measure of the amount of matter in an object. It is often measured in kilograms (kg) and is a fundamental property of an object.

2. How do you calculate the mass of an object?

The mass of an object can be calculated by dividing its weight (measured in Newtons) by the acceleration due to gravity (9.8 m/s^2 on Earth). This equation is known as Newton's Second Law of Motion: mass = weight / acceleration due to gravity.

3. What are combined pulleys?

Combined pulleys are a system of two or more pulleys that work together to distribute the force needed to lift an object. By using multiple pulleys, the force needed to lift an object is reduced, making it easier to lift heavy objects.

4. How do you determine the mechanical advantage of combined pulleys?

The mechanical advantage of combined pulleys can be determined by counting the number of strings supporting the object and adding one. For example, if there are two strings supporting the object, the mechanical advantage is 3 (2+1). This means that the force needed to lift the object is reduced by three times.

5. What is the difference between fixed and movable pulleys in a combined pulley system?

In a combined pulley system, fixed pulleys are attached to a fixed point, while movable pulleys are attached to the object being lifted. Fixed pulleys change the direction of the force, while movable pulleys reduce the amount of force needed to lift the object.

Similar threads

  • Introductory Physics Homework Help
Replies
3
Views
5K
Replies
10
Views
1K
  • Introductory Physics Homework Help
Replies
6
Views
3K
  • Introductory Physics Homework Help
Replies
10
Views
973
  • Introductory Physics Homework Help
Replies
4
Views
2K
  • Introductory Physics Homework Help
2
Replies
43
Views
3K
  • Introductory Physics Homework Help
Replies
15
Views
1K
  • Introductory Physics Homework Help
Replies
20
Views
2K
  • Introductory Physics Homework Help
Replies
5
Views
4K
  • Introductory Physics Homework Help
Replies
5
Views
10K
Back
Top